Diagonalizable Proof Homework: True or False?

  • Thread starter Thread starter metder
  • Start date Start date
  • Tags Tags
    Proof
Click For Summary
SUMMARY

The discussion centers on the diagonalizability of linear operators T and U defined on the space of n x n matrices over a field F, given a fixed n x n matrix A. It is established that if A is diagonalizable, then T(B) = AB is also diagonalizable. However, the claim that U(B) = AB - BA is diagonalizable when A is diagonalizable is false. The reasoning involves analyzing the effects of T and U on basis matrices when A is diagonal.

PREREQUISITES
  • Understanding of linear operators in vector spaces
  • Knowledge of diagonalizable matrices and their properties
  • Familiarity with matrix multiplication and commutation
  • Concept of basis matrices in linear algebra
NEXT STEPS
  • Study the properties of diagonalizable matrices in linear algebra
  • Learn about linear operators and their effects on vector spaces
  • Explore the implications of commutators in matrix theory
  • Investigate examples of linear operators and their diagonalizability
USEFUL FOR

Students of linear algebra, mathematicians focusing on matrix theory, and educators preparing coursework on diagonalizability and linear operators.

metder
Messages
5
Reaction score
0

Homework Statement



Let V be the space of n X n matrices over F. Let A be a fixed n X n matrix
over F. Let T and U be the linear operators on V defined by
T(B) = AB
U(B) = AB - BA.
1. True or false? If A is diagonalizable (over F), then T is diagonalizable.
2. True or false? If A is diagonalizable, then U is diagonalizable
Thanks for the help.



The Attempt at a Solution


I'm guessing that 1 is true and 2 is false. I'm not sure, since these are linear operators rather than simple matrices.
 
Physics news on Phys.org
Consider, first, the case in which A is diagonal. What do T and U do to the "basis" matrices?
 
Question: A clock's minute hand has length 4 and its hour hand has length 3. What is the distance between the tips at the moment when it is increasing most rapidly?(Putnam Exam Question) Answer: Making assumption that both the hands moves at constant angular velocities, the answer is ## \sqrt{7} .## But don't you think this assumption is somewhat doubtful and wrong?

Similar threads

  • · Replies 7 ·
Replies
7
Views
1K
  • · Replies 3 ·
Replies
3
Views
2K
  • · Replies 3 ·
Replies
3
Views
2K
  • · Replies 1 ·
Replies
1
Views
5K
  • · Replies 1 ·
Replies
1
Views
4K
  • · Replies 17 ·
Replies
17
Views
2K
  • · Replies 2 ·
Replies
2
Views
5K
  • · Replies 7 ·
Replies
7
Views
1K
  • · Replies 24 ·
Replies
24
Views
4K
  • · Replies 5 ·
Replies
5
Views
2K